Translate

Sunday, June 8, 2014

PTNN

Bài toán 1 (JBMO Shortlist 2008) : Tìm các số nguyên dương x,y,z sao cho : 4^{x}+3^{y}=z^{2}
Lời giải :
Phương trình tương đương :(z-2^{x})(z+2^{x})=3^{y}
Do đó :
\left\{\begin{matrix} z-2^{x}=3^{m} & & \\ z+2^{x}=3^{n} & & \end{matrix}\right.
Với m,n\in \mathbb{N},m<n,m+n=y
Trừ vế với vế : 3^{n}-3^{m}=2^{x+1}\Leftrightarrow 3^{m}(3^{n-m}-1)=2^{x+1}\Rightarrow \left\{\begin{matrix} m=0 & & \\ 3^{y}-1=2^{x+1} &(*) & \end{matrix}\right.
Giải phương trình  (*) :
Vì x\geq 1\Rightarrow 4|2^{x+1}\Rightarrow 3^{y}=2^{x+1}+1\equiv 1(mod4)\Rightarrow y chẵn
Đặt y=2k\qquad(k\in \mathbb{N}^{*})
Ta được (3^{k}-1)(3^{k}+1)=2^{x+1}\Rightarrow \left\{\begin{matrix} 3^{k}-1=2^{a} & & \\ 3^{k}+1=2^{b}& & \end{matrix}\right.
Với a<b;a+b=x+1;a,b\in \mathbb{N}
\Rightarrow 2^{b}-2^{a}=2\Rightarrow 2^{a}(2^{b-a}-1)=2\Rightarrow \left\{\begin{matrix} a=1 & & \\ b=2& & \end{matrix}\right.
Từ đó tìm được x=2y = 2z = 5
Kết luận : \boxed{(x;y;z)=(2;2;5)}

Bài toán 2: Tìm nghiệm nguyên dương  thỏa mãn phương trình sau:
                              
Lời giải:
   Xem phương trình trên như một phương trình bậc hai ẩn . Khi đó ta có:
              
   Để phương trình có nghiệm thì
                          
  Mặt khác, từ phương trình đã cho, dễ dàng suy ra:  
  Do đó, ta có  hoặc 
  Trường hợp 1  
                       hoặc  hoặc 
             1.1/      (loại).
             1.2/  .
             1.3/   (loại).
  Trường hợp 2:   
                        (loại).
  Kết luận: phương trình đã cho có nghiệm  
Bài toán 3 : Tìm các nghiệm nguyên dương của phương trình :
2^x=3^y+5.
Lời giải :
Xét y\leq 3, ta được cặp (x,y)=(3,1),(5,3). Xét y>3, kéo theo x>5. Ta sẽ chứng minh trong trường hợp này phương trình không còn nghiệm nguyên dương nào nữa.
Viết phương trình dưới dạng :
2^5(2^{x-5}-1)=3^3(3^{y-3}-1)
Ta có :
2^{x-5}\equiv 1\;(mod\;3^3)\Rightarrow 18=ord_{27}(2)\mid x-5
Lúc này :
19\mid 2^{18}-1\mid 2^{x-5}-1
Do vậy mà :
3^{y-3}\equiv 1\;(mod\;19)\Rightarrow 18=ord_{19}(3)\mid y-3
Suy ra :
2^5(2^{x-5}-1)\;\vdots \;3^{y-3}-1\;\vdots \;3^{18}-1\;\vdots\; 757
Kéo theo :
2^{x-5}\equiv 1\;(mod\;757)\Rightarrow 756=ord_{757}(2)\mid x-5\Rightarrow 54\mid x-5
Từ đó ta suy ra điều mâu thuẫn :
3^3(3^{y-3}-1)=2^5(2^{x-5}-1)\;\vdots \;2^{54}-1\;\vdots \;3^4
Đáp số của bài toán là (x,y)=(3,1),(5,3)
Bài toán 4 (Romania Team Selection Test 1998)
 Tìm tất cả các số nguyên dương (x,n) sao cho :
x^n+2^n+1\mid x^{n+1}+2^{n+1}+1
Lời giải :
Ta có :
x^{n+1}+2^{n+1}+1=x(x^n+2^n+1)-2^n.x+1-x+2^{n+1}
Do đó kết hợp với điều kiện bài toán, ta suy ra :
x^n+2^n+1\mid 2^{n+1}-2^nx+1-x
Với x=1,x=2 ta dễ dàng chỉ ra được điều mâu thuẫn. Ta chỉ xét x\geq 3. Khi đó không khó để thấy :
2^{n+1}-2^nx+1-x< 0
Từ đó mà :
2^nx-2^{n+1}+x-1=\left | 2^{n+1}-2^n.x+1-x \right |\geq x^n+2^n+1
Thu gọn thành :
(2^n+1)x\geq x^n+2^{n+1}+2^n+2\;\;\;(*)
Trước hết ta xét n\geq 3. Khi đó dễ dàng chứng minh được bằng quy nạp :
x^n\geq x(2^n+1)\Rightarrow x^n+2^{n+1}+2^n+2> x(2^n+1)
Mâu thuẫn với kết quả đã thu được ở trên. Do vậy phải có n\leq 2.
Nếu n=2 thì từ (*) có 5x\geq x^2+14, không tồn tại x.
Nếu n=1 thay vào giả thiết ban đầu :
x+3\mid x^2+5\Rightarrow x\in \left \{ 4,11 \right \}
Đáp số bài toán là :
Có duy nhất hai bộ số thỏa mãn đề bài là (x,n)=(4,1),(11,1)
Bài toán 5 (Vietnamese Mathematical Olympiad 2004)
Tìm bộ ba các số nguyên dương (x,y,z) thỏa mãn phương trình :
(x+y)(1+xy)=2^z
Lời giải :
Từ phương trình ta được \left\{\begin{matrix} x+y=2^a\\ 1+xy=2^b \end{matrix}\right.(a,b\in \mathbb{N}^*).
Theo định lí Viete thì x,y là hai nghiệm của phương trình :
X^2-2^a.X+2^b-1=0\;\;\;(*)
Để nghiệm của (*) nguyên thì biệt thức \Delta '=2^{2a-2}-2^b+1 phải là một số chính phương. Ta đặt:
2^{2a-2}-2^b+1=k^2\;\;(k\in \mathbb{N})\;\;(**)
  • Nếu 2^{b}\geq 2^{2a-2}. Ta cũng có 2^{2a}=(x+y)^2\geq 4xy=4(2^b-1)\Rightarrow 2^{2a}+1\geq 2^b.
Như vậy thì 2^{2a-2}\leq 2^{b}\leq 2^{2a-2}+1.
Khi 2^{b}=2^{2a-2} thì k=1. Khi ấy (*) có nghiệm X=2^{a-1}\pm 1. Từ đó (x,y)=(2^{a-1}-1,2^{a-1}+1),(2^{a-1}+1,2^{a-1}-1). Thế vào phương trình ban đầu tìm được z=3a-2.
Khi 2^{b}=2^{2a-2}+1, không mấy khó khăn ta tìm được (a,b)=(1,1). Từ đó thu được bộ (x,y,z)=(1,1,2).
  • Nếu 2^{b}< 2^{2a-2} thì viết (**) dưới dạng :
2^b(2^{2a-b-2}-1)=(k-1)(k+1)
Trường hợp a=1 hoặc b=1 đơn giản, cũng cho bộ (x,y,z)=(1,1,2). Ta chỉ xét a,b>1.
Khi đó dễ thấy k lẻ, từ đó kéo theo \gcd(k-1,k+1)=2.
Suy ra trong hai số k-1,k+1 sẽ có một số chia hết cho 2^{b-1}. Ta giả sử :
k=2^{b-1}m+\epsilon \;\;(\epsilon =\pm 1)
Thay ngược vào (**) :
2^b(2^{2a-b-2}-1)=k^2-1=2^{2b-2}m^2+2^bm\epsilon \Rightarrow 2^{2a-b-2}-1=2^{b-2}m^2+m\epsilon
Nhận thấy rằng có
(x-1)(y-1)\geq 0\Rightarrow xy+1\geq x+y\Rightarrow 2^b\geq 2^a\Rightarrow b\geq a.
Nếu b=a thì x=1 hoặc y=1. Ta tìm được bộ :
(x,y,z)=(1,2^a-1,2a),(2^a-1,1,2a)
Nếu b>a thì b-2>2a-b-2. Ta tiếp tục lập luận :
-m\epsilon -1=2^{b-2}m^2-2^{2a-b-2}=2^{2a-b-2}(2^{2b-2a}m^2-1)\geq m^2-1\Rightarrow m^2\pm m\leq 0
Các giá trị tìm được của m đều mâu thuẫn vì m nguyên dương.
Tổng hợp lại các kết quả, ta kết luận đáp số bài toán là :
(x,y,z)=(2^a-1,1,2a),(1,2^a-1,2a),(2^a-1,2^a+1,3a-2),(2^a+1,2^a-1,3a-2)
Trong đó a là số nguyên dương tùy ý.
Bài toán 6 (Indian National Olympiad 2008)
Tìm bộ ba (p,x,y) thỏa mãn phương trình :
p^x=y^4+4
Trong đó p là số nguyên tố và x,y là các số tự nhiên.
Lời giải :
Xét x=0 không thỏa mãn. Ta chỉ xét x\geq 1.
Phương trình trên dẫn đến hệ sau :
\left\{\begin{matrix} y^2-2y+2=p^m\\ y^2+2y+2=p^n \end{matrix}\right.(m,n\in \mathbb{N},m< n,m+n=x\geq 1)
Nếu m=0 thì được y=1, từ đó tìm được p=5,x=1.
Nếu n=0 thì y=-1, mâu thuẫn điều kiện y tự nhiên.
Do đó ta chỉ xét m,n\geq 1. Từ hệ trên ta suy ra :
p\mid (y^2+2y+2)-(y^2-2y+2)=4y.
Nếu p\mid 4 thì p=2. Nếu p\mid y thì do p\mid y^2-2y+2 nên p\mid 2. Tóm lại ta được p=2.
Thay vào phương trình ban đầu, ta được :
y^4+4=2^x
Nếu x=2 thì y=0. Xét x\geq 3. Ta được :
v_2(y^4)=v_2(2^x-4)=v_2(4(2^{x-2}-1))=2.
Đây là điều mâu thuẫn vì v_2(y^4)\geq 4. Đáp số của bài toán là (p,x,y)=(5,1,1),(2,2,0)
Bài toán 7 (Germany Team Selection Test 2010)
Tìm các số nguyên dương m,n thỏa mãn :
3^n-7^m=2
Lời giải :
Xét n=1, không tìm được m thỏa mãn. Xét n=2 thì được m=1. Xét m\geq 3 kéo theo n\geq 2.
Viết phương trình dưới dạng :
9(3^{m-2}-1)=7(7^{n-1}-1)
Từ phương trình trên ta được :
7^{n-1}\equiv 1\;(mod\;9)\Rightarrow 3=ord_9(7)\mid n-1
Từ đó ta được :
7^{n-1}-1\;\vdots \;7^3-1\;\vdots \;19
Kéo theo :
3^{m-2}\equiv 1\;(mod\;19)\Rightarrow 18=ord_{19}(3)\mid m-2
Lại suy ra được :
3^{m-2}-1\;\vdots \;3^{18}-1\;\vdots \;37
Do vậy mà :
7^{n-1}\equiv 1\;(mod\;37)\Rightarrow 9=ord_{37}(7)\mid m-1
Dẫn đến :
3^2(3^{m-2}-1)\;\vdots \;7^{n-1}-1\;\vdots\; 7^9-1\;\vdots \;27
Đây là điều mâu thuẫn. Có duy nhất một cặp số thỏa mãn bài ra là (m,n)=(2,1)
Bài toán 8 (Japan Mathematical Olympiad Finals 2014)
Tìm tất cả các bộ ba (a,b,c) với a,b,c nguyên dương và thỏa mãn :
2^a+3^b+1=6^c
Lời giải :
Xét a=1 được 3^b+3=6^c. Nếu b=1 thì c=1. Nếu b>1 thì v_3(6^c)=v_3(3^b+3)=v_3(3(3^{b-1}+1))=1\Rightarrow c=1. Suy ra b=1, mâu thuẫn. Ta được bộ (1,1,1).
Xét a=2 được 3^b+5=6^c\equiv 0\;(mod\;3), mâu thuẫn.
Xét a\geq 3 thì :
6^c=2^a+3^b+1\equiv 2,4\;(mod\;8)\Rightarrow c\in \left \{ 1,2 \right \}.
Nếu c=1 thì 2^a+3^b=5, vô lí vì a \geq 3.
Nếu c=2 thì 2^a+3^b=35, thu được a=b=3.
Đáp số của bài toán là (a,b,c)=(1,1,1),(3,3,2)
Bài toán 9 (Turkey National Olympiad Second Round 2013)
Tìm tất cả các số nguyên dương m,n thỏa mãn :
2^n+n=m!
Lời giải :
Xét m\leq 6. Do 2^n+n\geq 3 nên m\geq 3. Như vậy ta xét m\in \left \{ 3,4,5,6 \right \}. Thử từng trường hợp ta được cặp (m,n)=(3,2).
Ta chỉ xét m>6. Đặt n=2^k.t\;\;\;\;(k\in \mathbb{N},t\in \mathbb{Z}^{+},2\nmid t). Dễ thấy n>k.
Ta có :
m!=2^n+n=2^n+2^k.t=2^k(2^{n-k}+t)
Suy ra :
k=v_2(2^k.(2^{n-k}+t))=v_2(m!)
Lại theo định lí Legendre ta được :
k=v_2(m!)=\underset{i=1}{\overset{+\infty }{\sum}} \left \lfloor \dfrac{m}{2^i} \right \rfloor\geq \left \lfloor \dfrac{m}{2} \right \rfloor
Mặt khác ta cũng dễ dàng chứng minh được bằng quy nạp rằng với m>6 thì :
2^{\left \lfloor m/2 \right \rfloor}\geq m và 2^{m^2}> m!
Nếu mà m\geq t thì t\mid m!=2^n+n=2^n+2^k.t\Rightarrow t\mid 2^n, mâu thuẫn vì t lẻ,
Do đó phải có t>m. Từ đó dẫn đến :
n=2^k.t\geq 2^{\left \lfloor m/2 \right \rfloor}.m> m^2\Rightarrow 2^n> 2^{m^2}> m!
Phương trình không có nghiệm nguyên dương.
Đáp số duy nhất của bài toán là (m,n)=(3,2)
Bài toán 10 (Finland Finish National High School Mathematics Competition 2013)
Tìm tất cả các bộ ba số nguyên (m,p,q) thỏa mãn :
2^m.p^2+1=q^5
Trong đó m>0 và p,q là hai số nguyên tố.
Lời giải :
Nếu p=2 thì :
2^{m+1}+1=q^5\Leftrightarrow (q-1)(q^4+q^3+q^2+q+1)=2^{m+1}
Vì q lẻ nên q^4+q^3+q^2+q+1 lẻ, suy ra 2^{m+1} có ước lẻ, vô lí.
Xét p lẻ. Ta viết phương trình dưới dạng :
(q-1)(q^4+q^3+q^2+q+1)=2^m.p^2
Do q^4+q^3+q^2+q+1 lẻ nên q^4+q^3+q^2+q+1=p hoặc q^4+q^3+q^2+q+1=p^2.
Nhưng rõ ràng không thể xảy ra trường hợp q^4+q^3+q^2+q+1=p vì khi ấy thì q-1=2^m.p. Do 2^m.p>p nên q-1>q^4+q^3+q^2+q+1 và đây là điều mâu thuẫn.
Như vậy chỉ có thể là :
q^4+q^3+q^2+q+1=p^2
Ta có thể dễ dàng lập luận được rằng :
(2q^2+q)^2< 4p^2< (2q^2+q+2)^2
Từ đó suy ra 4p^2=(2q^2+q+1)^2
Ta được phương trình :
4(q^4+q^3+q^2+q+1)=(2q^2+q+1)^2\Leftrightarrow q^2-2q-3=0\Rightarrow q=3
Từ đó tìm được p=11,m=1.
Đáp số bài toán là :
m=1,p=11,q=3
Bài toán 11 (Indian National Olympiad 2013)
Tìm tất cả các số tự nhiên m,n và số nguyên tố p\geq 5 thỏa mãn :
m(4m^2+m+12)=3(p^n-1)
Lời giải :
Ta đưa phương trình về dạng :
(4m+1)(m^2+3)=3p^n
Nếu n=0 thì (4m+1)(m^3+3)=3, ta được m=0.
Nếu n=1 thì (4m+1)(m^3+3)=3p. Dễ thấy loại trường hợp này.
Ta chỉ xét n\geq 2, do m=0 không thỏa nên ta cũng xét m>0.
Do 4m+1>3 và m^3+3>3 nên cả hai số này phải cùng chia hết cho p. Ta có :
p\mid m(4m+1)-4(m^2+3)\Rightarrow p\mid m-12\Rightarrow p\mid 4m-28\Rightarrow p\mid (4m+1)-49\Rightarrow p\mid 49\Rightarrow p=7
Ta được phương trình :
(4m+1)(m^2+3)=3.7^n
Có hai khả năng xảy ra. Ta xét khả năng thứ nhất là :
\left\{\begin{matrix} 4m+1=7^x\\ m^2+3=3.7^y \end{matrix}\right.\;(x,y\geq 1;x,y\in \mathbb{N})
Khử m từ hai phương trình :
\left ( \dfrac{7^x-1}{4} \right )^2+3=3.7^y\Leftrightarrow (7^x-1)^2+48=48.7^y\Leftrightarrow 7^{2x}-2.7^x+49=48.7^y
Gỉa sử rằng x,y>2 thì kéo theo :
7^{x-2}-2.7^{x-2}+1=48.7^{y-2}
Đẳng thức này là không thể xảy ra.
Do vậy ta suy ra hoặc x\leq 2 hoặc y\leq 2. Thử trực tiếp ta được m=12 ứng với x=y=2.
Tiếp theo xét khả năng thứ hai :
\left\{\begin{matrix} 4m+1=3.7^x\\ m^2+3=7^y \end{matrix}\right.(x,y\geq 1;x,y\in \mathbb{N})
Hoàn toàn tương tự khả năng trên, nhưng trường hợp này không cho nghiệm.
Đáp số bài toán là :
m=n=0p\geq 5 là số nguyên tố tùy ý.
m=12,n=4,p=7
Bài toán 12v (Japan Mathematical Olympiad Finals 2009)
Tìm tất cả các số nguyên dương n sao cho 8^n+n chia hết cho 2^n+n.
Lời giải :
Ta có 2^n+n\mid 8^n+n^3, lại theo đề bài thì 2^n+n\mid 8^n+n. Dẫn đến :
2^n+n\mid n^3-n
Ta chứng minh bằng quy nạp rằng nếu n\geq 10 thì :
2^n> n^3\;\;\;\;\;\;(*)
Gỉa sử có (*). Ta cần chứng minh 2^{n+1}> (n+1)^3. Do có (*) nên ta chỉ cần chứng minh :
2n^3> (n+1)^3\Leftrightarrow n^3> 3n^2+3n+1\Leftrightarrow n^2(n-9)+n(n^2-9)+(n^3-3)> 0
Và vì n\geq 10 nên điều này luôn đúng. Theo nguyên lí quy nạp ta có :
2^{n}> n^3,\;\forall n\in \mathbb{N},n\geq 10
Kéo theo 2^n+n> n^3-n. Thế nhưng lại có 2^n+n\mid n^3-n\Rightarrow n^3-n\geq 2^n+n. Hai điều này mâu thuẫn nhau.
Từ đó ta có n\leq 9. Thử trực tiếp ta được đáp số của bài toán là :
n\in \left \{ 1,2,4,6 \right \}

No comments: